Completed rec9 solutions
[course.git] / latex / problems / Young_and_Freedman_12 / problem28.62.tex
1 \begin{problem*}{28.62}
2 A pair of long, rigid metal rods, each of length $L$, lie parallel to
3 each other on a perfectly smooth table.  Their ends are connected by
4 identical, very light conducting springs of force constant $k$
5 (Fig.~28.55) and negligable unstretched length.  If a current $I$ runs
6 through this circuit, the springs will stretch.  At what seperation
7 will the rods remain at rest?  Assume that $k$ is large enough so that
8 the separation of the rods will be much less than $L$.
9 \begin{center}
10 \begin{asy}
11 import Mechanics;
12
13 real u = 1cm;
14 real L = 4u;
15 real d = 1u;
16 real Isep = 6pt;
17 real hBar = 1mm;
18 pen ipen = red+blue;
19
20 Block Bt = Block((0,0), width=L, height=hBar, fill=yellow);
21 Block Bb = Block((0,-d), width=L, height=hBar, fill=yellow);
22 Spring Sl = Spring((-L/2,-d), (-L/2,0),
23                    deadLength=1mm, unstretchedLength=d, "k");
24 Spring Sr = Spring((L/2,0), (L/2,-d),
25                    deadLength=1mm, unstretchedLength=d, "k");
26 Vector It = Vector((0,Isep), dir=0, ipen);
27 Vector Ib = Vector((0,-d-Isep), dir=180, ipen);
28
29 Distance DL = Distance((-L/2,0), (L/2,0), offset=-2Isep, "$L$");
30 Distance Dd = Distance((L/2,0), (L/2,-d), offset=-4Isep, "$d$");
31
32 Sl.draw(rotateLabel=false);
33 Sr.draw(rotateLabel=false);
34 Bt.draw();
35 Bb.draw();
36 It.draw();
37 Ib.draw();
38 DL.draw();
39 Dd.draw(rotateLabel=false);
40 \end{asy}
41 \end{center}
42 \end{problem*}
43
44 \begin{solution}
45 In order for the rods to remain at rest, the net force on each rod
46 must be zero.  The only forces we need to consider are the spring
47 forces $F_s$ and magnetic forces $F_B$.  The only other possible force
48 for this problem would be a gravitational force $F_g=mg$, but no mass
49 $m$ is given for the rods, so we must assume $F_g$ is negligable.
50
51 Because of Newton's third law, we know that if the forces of the
52 bottom rod on the top rod cancel, then the forces from the top rod on
53 the bottom rod must also cancel, and we can restrict our force
54 balancing to only the top rod.
55
56 The spring force on the top rod is
57 \begin{equation}
58   F_s = 2kd
59 \end{equation}
60 downward, due to a $kx$ force for each spring (left and right), with
61 $x=d$ because the unstetched length of the springs is negligable.
62
63 The magnetic force on the top rod is
64 \begin{equation}
65   \vect{F}_B = I\vect{L}\times\vect{B} \;,
66 \end{equation}
67 where \vect{B} is the magnetic field along the top rod created by the
68 current in the bottom rod.  This magnetic field can be approximated as
69 as that due to an infinitely long, straight wire a distance $d$ below
70 the top wire
71 \begin{equation}
72   B = \frac{\mu_0 I}{2\pi d}
73 \end{equation}
74 directed into the board.  The bottom wire is not infinitely long, but
75 because $d\ll L$, fringe effects due to the wire's finite length are
76 small.  Because the magnetic field is into the board and the current
77 in the top wire is to the right, the vector $\vect{F}_B$ is
78 \begin{equation}
79   F_B = |I\vect{L}|\cdot|\vect{B}|\sin\theta = ILB
80     = \frac{\mu_0 I^2L}{2\pi d}
81 \end{equation}
82 directed upward.
83
84 Putting this all together to balance the forces on the top bar
85 \begin{align}
86   0 &= \sum{F_y} = F_B - F_s = \frac{\mu_0 I^2L}{2\pi d} - 2kd \\
87   2kd &= \frac{\mu_0 I^2L}{2\pi d} \\
88   d^2 &= \frac{\mu_0 I^2L}{4\pi k} \\
89   d &= \ans{\sqrt{\frac{\mu_0 L}{4\pi k}}\;I}\;.
90 \end{align}
91 \end{solution}